[obm-l] Re: [obm-l] Re: [obm-l] Re: [obm-l] Re: [obm-l] Dúvida conceitual (equações)

2018-10-15 Por tôpico Bernardo Freitas Paulo da Costa
On Mon, Oct 15, 2018 at 8:07 AM Claudio Buffara
 wrote:
>
> Derivando e igualando a zero o lado esquerdo da sua equação, ficamos com:
> -2*cos(x)*sen(x) + sen(x) = 0 ==>
> sen(x) = 0  ou  cos(x) = 1/2 ==>
> x = 0 ou x = pi ou x = 2pi
> ou x = pi/3 ou x = 5pi/3.
>
> Assim, uma definição que me parece adequado para equações em geral (e não 
> necessariamente polinomiais) da forma f(x) = 0 é que uma raiz de 
> multiplicidade n é raiz de f, f’, ... , f^(n-1) mas não é raiz de f^(n).
>
> Naturalmente, se f não tiver todas as derivadas, precisaremos achar uma 
> definição diferente. Mas talvez, neste caso, nem faça sentido falar em 
> multiplicidade de uma raiz.

Essa definição funciona relativamente bem se f é analítica, porque o
comportamento local é determinado por inteiros.  Se f for apenas
diferenciável, talvez seja complicado dizer algo, como o exemplo
clássico de exp(-1/x^2).  A raiz tem multiplicidade infinita?

Enfim, existem, como você falou, boas razões para incorporar
multiplicidade (por exemplo estabilidade numérica), mas isso em geral
só faz sentido no mundo analítico, onde a noção de "grau" é dada pelas
derivadas.  Acho que mesmo no mundo C-infinito já pode haver
problemas, mas não sou especialista (nessas :D) patologias.  A questão
original, incluindo multiplicidades, pode ser resolvida simplesmente
usando as relações de Girard, que dependem de forma simples da
equação.

Vou tentar dar um exemplo que ilustra meu ponto de vista:  qual o
produto das raízes da equação x^2 - 4x + c?  "Qualquer um" dirá "c".
Mas, naturalmente, se c = 4, a única solução é x=2, e portanto (sem
usar multiplicidades) este produto seria apenas 2.  E daí a fórmula
fica muito mais complicada, com um caso especial, e descontínua.  A
grande sacada do Girard foi, justamente, propor incorporar as
multiplicidades, para simplificar as fórmulas (além, é claro, de
incluir também as soluções negativas, antes consideradas como
"absurdas" - este foi, provavelmente, o maior motivo de as pessoas
considerarem raízes negativas como algo que fazia sentido, e portanto
os números negativos também).  Mas isso não quer dizer que a equação
x^2 - 4x + 4 tenha duas soluções.  É apenas uma forma mais conveniente
de interpretar as raízes quando se pensam nas relações de Girard (e
várias outras fórmulas).  Neste sentido, acho que este tipo de questão
mais atrapalha (porque "era só para usar a fórmula") - a menos que,
justamente, se discuta *porque* falamos de multiplicidade: para que as
fórmulas fiquem mais simples (e você pode incluir "bonitas" também,
por minha conta).  Nada mais.  E esta "simplificação" do entendimento
através da simplificação das fórmulas não se justifica sempre: este
mesmo debate sobre multiplicidades leva a considerar objetos no
infinito (para que todas as retas se intersectem sempre em um ponto),
complexos (para x^2 + 1 = 0 ter raiz), etc.  Muitas vezes, é útil ter
esse entendimento unificado, onde tudo "só depende do grau".  Mas será
mesmo que se eu perguntar para você "em quantos pontos a reta x=3
corta a parábola y=x^2?" você vai dizer "2, é óbvio"?

Abraços,
-- 
Bernardo Freitas Paulo da Costa

-- 
Esta mensagem foi verificada pelo sistema de antiv�rus e
 acredita-se estar livre de perigo.


=
Instru��es para entrar na lista, sair da lista e usar a lista em
http://www.mat.puc-rio.br/~obmlistas/obm-l.html
=


[obm-l] Re: [obm-l] Re: [obm-l] Re: [obm-l] Re: [obm-l] Dúvida num Enunciado

2018-04-26 Por tôpico Pedro José
Boa tarde!
Bernardo,
Realmente eu falhei. Fiquei com a expressão |x+3| < 4 na cabeça. Até uso um
delta, e comento que não pode ser maior que 4.
Saudações,
PJMS

Em 25 de abr de 2018 22:33, "Jaare Oregim" 
escreveu:

>
>
> 2018-04-25 21:30 GMT-03:00 Bernardo Freitas Paulo da Costa <
> bernardo...@gmail.com>:
>
>> 2018-04-25 20:41 GMT-03:00 Claudio Buffara :
>> > O [...]
>> "Determine r > 0 tal que [ |x+3| < r => (A^2 - 10A + 9 > 0 para todo A
>> real) ]."
>>
>> Que continua com o "problema" de ter um "x" livre.  Daí, a proposição
>> entre colchetes tem um valor (verdadeiro/falso) que depende de x.
>>
>
> se o x tá livre *não* tem valor-verdade. Sentença aberta não tem
> valor-verdade.
>
> tb acho que a intenção é "Determine r > 0 tal que (para todo x real, |x+3|
> < r => x^2 - 10x + 9 > 0)."
>
> --
> Esta mensagem foi verificada pelo sistema de antivírus e
> acredita-se estar livre de perigo.

-- 
Esta mensagem foi verificada pelo sistema de antiv�rus e
 acredita-se estar livre de perigo.



[obm-l] Re: [obm-l] Re: [obm-l] Re: [obm-l] Re: [obm-l] Dúvida num Enunciado

2018-04-25 Por tôpico Claudio Buffara
Verdade! Reparei agora que deve ser r > 0.
Então provavelmente o "para todo x real" não deveria estar lá.
Neste caso, vira um problema com mais cara de EM:

Achar todos os r > 0 tais que
SE  x pertence ao intervalo (-3-r , -3+r )
ENTÃO  x^2 - 10x + 9 > 0

x^2 - 10x + 9 > 0  sss  x pertence a (-inf,1) união (9,+inf).

Assim, observamos que se 0 < r <= 4, então (-3-r,-3+r) está contido em
(-inf,1).
Logo, se 0 < r <= 4 então a implicação acima é verdadeira.

[]s,
Claudio.




2018-04-25 21:47 GMT-03:00 Pedro José :

> Boa noite!
> Cláudio,
> o problema tem restrição r>0. Não dá para seguir nessa linha de r< 0.
> Saudações,
> PJMS
>
> Em 25 de abr de 2018 21:42, "Bernardo Freitas Paulo da Costa" <
> bernardo...@gmail.com> escreveu:
>
>> 2018-04-25 20:20 GMT-03:00 Pedro José :
>> > Boa tarde!
>> > Realmente o enunciado está mal feito.
>> >
>> > Se |x+3| < r, não pode ser para todo o Real. Na verdade é x pertence a
>> |R.
>> >
>> > x^2 -10x + 9 >0  ==> x pertence a A = (-oo, 1) U (9,oo)
>> >
>> > então temos que escolher r de modo que quando resolvamos |x + 3| < r,
>> tenha
>> > x num subconjunto de A
>> >
>> > x < -3 ==> x+3 < 0 ==> -x -3 < r ==> r > x+3 Se r > 4 vai ter 1=< x =<9
>> > atendendo |x +3| <4 + delta. Portanto x <4
>> > então |x+3| < 4, conferindo
>> > x > -3 ==> x+3 <4  ==> x<1, atende.
>> > se x<-3 atende por hipótese. Mas se quiser conferir. -x - 3 < 4 : -x <
>> 7: x
>> >>7, mas x <-3, não tem solução.
>> >
>> > x>=- 3 ==> x+3>=0 ==> x+3 < r. Se r >=4, existirá solução em [1,9].
>> >
>> > Portanto r pertence a (0,4)
>>
>> Só um detalhe: r = 4 também serve: se |x+3| < 4, temos -7 < x < 1, que
>> está contido em A.
>>
>> A minha forma preferida de resolver este exercício é gráfica:
>> desenhamos o conjunto A, depois tomamos P = -3, e traçamos um
>> intervalo simétrico em P de maior raio possível contido em A.  Dá r <=
>> 4 ou r < 4 (no desenho, é difícil decidir entre o estrito ou não) e
>> daí tem que pensar um pouco para detectar se r = 4 serve.
>>
>> Abraços,
>> --
>> Bernardo Freitas Paulo da Costa
>>
>> --
>> Esta mensagem foi verificada pelo sistema de antivírus e
>>  acredita-se estar livre de perigo.
>>
>>
>> =
>> Instru�ões para entrar na lista, sair da lista e usar a lista em
>> http://www.mat.puc-rio.br/~obmlistas/obm-l.html
>> =
>>
>
> --
> Esta mensagem foi verificada pelo sistema de antivírus e
> acredita-se estar livre de perigo.
>

-- 
Esta mensagem foi verificada pelo sistema de antiv�rus e
 acredita-se estar livre de perigo.



[obm-l] Re: [obm-l] Re: [obm-l] Re: [obm-l] Re: [obm-l] Dúvida num Enunciado

2018-04-25 Por tôpico Luiz Antonio Rodrigues
Olá, Bernardo!
Boa noite!
Vou tentar fazer a resolução graficamente...
Muito obrigado!
Um abraço!
Luiz

On Wed, Apr 25, 2018, 9:55 PM Pedro José  wrote:

> Boa noite!
> Cláudio,
> o problema tem restrição r>0. Não dá para seguir nessa linha de r< 0.
> Saudações,
> PJMS
>
> Em 25 de abr de 2018 21:42, "Bernardo Freitas Paulo da Costa" <
> bernardo...@gmail.com> escreveu:
>
>> 2018-04-25 20:20 GMT-03:00 Pedro José :
>> > Boa tarde!
>> > Realmente o enunciado está mal feito.
>> >
>> > Se |x+3| < r, não pode ser para todo o Real. Na verdade é x pertence a
>> |R.
>> >
>> > x^2 -10x + 9 >0  ==> x pertence a A = (-oo, 1) U (9,oo)
>> >
>> > então temos que escolher r de modo que quando resolvamos |x + 3| < r,
>> tenha
>> > x num subconjunto de A
>> >
>> > x < -3 ==> x+3 < 0 ==> -x -3 < r ==> r > x+3 Se r > 4 vai ter 1=< x =<9
>> > atendendo |x +3| <4 + delta. Portanto x <4
>> > então |x+3| < 4, conferindo
>> > x > -3 ==> x+3 <4  ==> x<1, atende.
>> > se x<-3 atende por hipótese. Mas se quiser conferir. -x - 3 < 4 : -x <
>> 7: x
>> >>7, mas x <-3, não tem solução.
>> >
>> > x>=- 3 ==> x+3>=0 ==> x+3 < r. Se r >=4, existirá solução em [1,9].
>> >
>> > Portanto r pertence a (0,4)
>>
>> Só um detalhe: r = 4 também serve: se |x+3| < 4, temos -7 < x < 1, que
>> está contido em A.
>>
>> A minha forma preferida de resolver este exercício é gráfica:
>> desenhamos o conjunto A, depois tomamos P = -3, e traçamos um
>> intervalo simétrico em P de maior raio possível contido em A.  Dá r <=
>> 4 ou r < 4 (no desenho, é difícil decidir entre o estrito ou não) e
>> daí tem que pensar um pouco para detectar se r = 4 serve.
>>
>> Abraços,
>> --
>> Bernardo Freitas Paulo da Costa
>>
>> --
>> Esta mensagem foi verificada pelo sistema de antivírus e
>>  acredita-se estar livre de perigo.
>>
>>
>> =
>> Instru�ões para entrar na lista, sair da lista e usar a lista em
>> http://www.mat.puc-rio.br/~obmlistas/obm-l.html
>> =
>>
>
> --
> Esta mensagem foi verificada pelo sistema de antivírus e
> acredita-se estar livre de perigo.

-- 
Esta mensagem foi verificada pelo sistema de antiv�rus e
 acredita-se estar livre de perigo.



[obm-l] Re: [obm-l] Re: [obm-l] Re: [obm-l] Re: [obm-l] Dúvida em uma solução (conjunto denso)

2017-07-10 Por tôpico Pedro Soares
Sim, é uma prova por absurdo.

''...o autor parte de uma hipótese contrária ao resultado pra chegar num
absurdo...''

2017-07-11 1:03 GMT-03:00 Bernardo Freitas Paulo da Costa <
bernardo...@gmail.com>:

> 2017-07-10 18:56 GMT+03:00 Antonio Carlos :
> > Entendi. Muito obrigado, Pedro!
>
> Tem um problema muito sério, que os logs são diferentes...
>
> log_2 3 = log(3)/log(2) = 1.5849625007211563
> log_3 6 = log(6)/log(3) = 1.6309297535714573
>
> Mas o problema está, provavelmente, na primeira hipótese (que ela
> também é falsa).  A demonstração por densidade está certa, e talvez
> seja no meio de um raciocínio por absurdo, mas sei lá...
> --
> Bernardo Freitas Paulo da Costa
>
> --
> Esta mensagem foi verificada pelo sistema de antivírus e
>  acredita-se estar livre de perigo.
>
>
> =
> Instru�ões para entrar na lista, sair da lista e usar a lista em
> http://www.mat.puc-rio.br/~obmlistas/obm-l.html
> =
>

-- 
Esta mensagem foi verificada pelo sistema de antiv�rus e
 acredita-se estar livre de perigo.



[obm-l] Re: [obm-l] Re: [obm-l] Re: [obm-l] Re: [obm-l] Dúvida sobre a Obm U

2016-07-26 Por tôpico Carlos Gomes
Um bom livro é Razvan Gelca, Titu Andreescu-Putnam and Beyond (2007)

Cgomes.

Em 26 de julho de 2016 08:57, Otávio Araújo 
escreveu:

> Não, onde posso conseguir? e do que ela trata?
>
> Em 25 de julho de 2016 11:32, Carlos Victor 
> escreveu:
>
>>
>>
>>
>> Oi Otávio,
>>
>> Você já viu a Revista Matemática Universitária da SBM ?
>>
>> Em 25/07/2016 10:09, Otávio Araújo escreveu:
>>
>>
>>
>> Pois é, se algum professor com experiência em olimpíadas, como o Nicolau
>> por exemplo, respondesse minha pergunta seria de grande ajuda
>>
>> Em 24 de jul de 2016, às 23:25, Israel Meireles Chrisostomo <
>> israelmchrisost...@gmail.com> escreveu:
>>
>> Boa pergunta, eu também tenho interesse em participar da OBM U e
>> gostaria de umas dicas
>>
>> Em 16 de julho de 2016 13:29, Otávio Araújo 
>> escreveu:
>>
>>> Galera, gostaria que vocês me dessem dicas de o que estudar, como
>>> estudar e por quais livros e materiais estudar para a prova da Obm nível
>>> universitário...
>>> Estou muito interessado em participar, mas fico meio confuso por onde
>>> estudar...
>>> Por favor me ajudem
>>> --
>>> Esta mensagem foi verificada pelo sistema de antivírus e
>>> Â acredita-se estar livre de perigo.
>>>
>>>
>>> =
>>> Instruções para entrar na lista, sair da lista e usar a lista em
>>> http://www.mat.puc-rio.br/~obmlistas/obm-l.html
>>> =
>>>
>>
>> --
>> Esta mensagem foi verificada pelo sistema de antivírus e
>> acredita-se estar livre de perigo.
>>
>>
>> --
>> Esta mensagem foi verificada pelo sistema de antivrus e
>> acredita-se estar livre de perigo.
>>
>>
>> --
>> Esta mensagem foi verificada pelo sistema de antivírus e
>> acredita-se estar livre de perigo.
>>
>
>
> --
> Esta mensagem foi verificada pelo sistema de antivírus e
> acredita-se estar livre de perigo.
>

-- 
Esta mensagem foi verificada pelo sistema de antiv�rus e
 acredita-se estar livre de perigo.



[obm-l] Re: [obm-l] Re: [obm-l] Re: [obm-l] Re: [obm-l] Dúvida sobre a Obm U

2016-07-25 Por tôpico Tiago Sandino
Égua ma, sou mais ou menos da UFC, de qualquer forma, começar matemática
UFC prox ano. Fiz olimpíada um tempo, imergi totalmente nisso. Fiz e
trabalhei com engenharia elétrica uns anos, larguei o curso no final pq o
negócio na engenharia era próprio e precisava de tempo. Atualmente tô dando
aula de turma ITA IME e olimpíada de mat no Colégio Militar. Não sei se eu
vou poder fazer a prova da OBMU, mas estudarei. E a olimpíada universitária
é uma continuação da não universitária, daí meu interesse, já que sou
professor disso.
Precisando de dica, ta aí meu contato. 85 9 99134896. Se tem uma coisa que
eu sei sobre olimpíada é que o cara tem que tá no meio, tem que falar sobre
isso, tem que conhecer pessoas do meio etc. Só assim vc evolui. Só assim vc
passa de ser um cara que consegue aplicar fórmula a ser um cara que "cria"
matemática todo dia, que na minha concepção é a maior realização pessoal na
olimpíada.


Em 25 de julho de 2016 17:19, Otávio Araújo 
escreveu:

>
>
> Égua Tiago, eu também sou do Ceará mas meu celular atualmente não tem
> chip Mas tu é da UFC Tiago? E ainda estou esperando algum professor com
> experiência em olimpíadas de matemática responder a minha pergunta
>
> Em 25 de jul de 2016, às 13:38, Tiago Sandino 
> escreveu:
>
> Oi pessoal.
> Tem diversos livros de olimpíadas para graduandos (undergrads) ou com
> capítulos de temas exclusivamente (até onde eu saiba) universitários.
> Grátis na net, que eu saiba, tem muita coisa no AOPS. Dois links aqui:
> 1) *Fórum*: https://www.artofproblemsolving.com/community/c7_college_math
> 2) *Fórum por Competições*:
> https://www.artofproblemsolving.com/community/c15_undergraduate_contests
>
> Sou do Ceará, tava meio afastado da Matemática, mas fiz as pazes com ela
> recentemente. Se alguém quiser formar um grupo de estudo pelo Whatsapp...
> segue meu número: 85 9 9913 4896.
>
> Att.
> Tiago Sandino
>
> Em 25 de julho de 2016 10:20, Raul Alves  escreveu:
>
>> Também tenho interesse na OBMU, e a 1ª fase tá chegando.
>> Se algum professor puder organizar algum material de apoio, seria de
>> grande ajuda
>>
>> Em 25 de julho de 2016 10:09, Otávio Araújo 
>> escreveu:
>>
>>>
>>>
>>> Pois é, se algum professor com experiência em olimpíadas, como o
>>> Nicolau por exemplo, respondesse minha pergunta seria de grande ajuda
>>>
>>> Em 24 de jul de 2016, Ã s 23:25, Israel Meireles Chrisostomo <
>>> israelmchrisost...@gmail.com> escreveu:
>>>
>>> Boa pergunta, eu também tenho interesse em participar da OBM U e
>>> gostaria de umas dicas
>>>
>>> Em 16 de julho de 2016 13:29, Otávio Araújo <
>>> otavio17.ara...@gmail.com> escreveu:
>>>
 Galera, gostaria que vocês me dessem dicas de o que estudar,
 como estudar e por quais livros e materiais estudar para a prova da Obm
 nível universitário...
 Estou muito interessado em participar, mas fico meio confuso por onde
 estudar...
 Por favor me ajudem
 --
 Esta mensagem foi verificada pelo sistema de antivírus e
  acredita-se estar livre de perigo.



 =
 Instruções para entrar na lista, sair da lista e usar a lista em
 http://www.mat.puc-rio.br/~obmlistas/obm-l.html

 =

>>>
>>>
>>> --
>>> Esta mensagem foi verificada pelo sistema de antivírus e
>>> acredita-se estar livre de perigo.
>>>
>>>
>>> --
>>> Esta mensagem foi verificada pelo sistema de antivírus e
>>> acredita-se estar livre de perigo.
>>>
>>
>>
>>
>> --
>> *Raul Lima Alves*
>>
>> *Estagiário na Aton Engenharia*
>> *Estudante de Engenharia de Computação - UFBA*
>> *Telefone: (71) 9103-0878*
>> *Facebook:Â *https://www.facebook.com/raul.alves.161
>> *LinkedIn*:Â https://br.linkedin.com/in/raul-alves-8b090228
>> 
>>
>>
>>
>> --
>> Esta mensagem foi verificada pelo sistema de antivírus e
>> acredita-se estar livre de perigo.
>>
>
>
> --
> Esta mensagem foi verificada pelo sistema de antivírus e
> acredita-se estar livre de perigo.
>
>
> --
> Esta mensagem foi verificada pelo sistema de antivírus e
> acredita-se estar livre de perigo.
>

-- 
Esta mensagem foi verificada pelo sistema de antiv�rus e
 acredita-se estar livre de perigo.



[obm-l] Re: [obm-l] Re: [obm-l] Re: [obm-l] Re: [obm-l] Dúvida

2011-11-02 Por tôpico Kleber Bastos
Achei que faltava a regra para f(x),mas a menina disse que não. Vou
verificar com a pessoa e retorno.
Obrigado!

Em 02/11/2011 23:08, Joao Maldonado joao_maldonad...@yahoo.com.br
escreveu:



Não tem como ser isso não cara
Traduz isso aí que   não dá pra entender

O que poderia ser é
Mostre que  qualquer que seja o número  racional e positivo  a/b com a e  b
inteiros  primos entre si,  é válido   que f(a/b) =  f(1)^(a/b)



Tudo bem,  vamos dizer  que é isto, mas qual a regra para  a função f(x)??

Não são  todas as funções f(x)  que  satisfazem tal  afirmação  (aliás,  a
única   que consegui pensar até agora é  f(x) = a^x

f(x) = x²,  f(4/7) = 16/49 !=   1^(4/7) = 1

[]'s

João



--
Date: Wed, 2 Nov 2011 17:17:59 -0200
Subject: [obm-l] Dúvida
From: klebe...@gmail.com
To: obm-l@mat.puc-rio.br



Olá grupo,
Estou me enrolando nesta prova.

Mostre q ∀ nº a/b0, MDC(a,b) = 1,
é válido: f(a/b) =...

--
*De:* Kleber Bastos klebe...@gmail.com
*Para:* obm-l@mat.puc-rio.br
*Enviadas:* Quarta-feira, 2 de Novembro de 2011 22:21
*Assunto:* [obm-l] Re: [obm-l] Re: [obm-l] Dúvida


É isso mesmo:
Mostrar que ∀ nº racional a/b0, M.D.C.(a,b)=1 é válida a sentença:
f(a/b)=f(1)^a/b (...


[obm-l] Re: [obm-l] Re: [obm-l] RE: [obm-l] Re: [obm-l] Re: [obm-l] dúvida sobre séries

2011-06-08 Por tôpico Artur Costa Steiner
O critério mais simples para mostrar que a série harmônica diverge talvez
seja o baseado no seguinte teorema:

Se x_n é uma sequência decrescente de reais tal que Soma x_n converge, então
lim n x_ n = 0. (Prove isto)

Se x_n = 1/n, x_n decresce para 0 mas lim n x_n = 1, o que mostra que Soma
x_n diverge. Para infinito, pois os termos são positivos.

Mas talvez não seja uma prova tão elucidativa quanto as outras dadas.

Artur

Artur Costa Steiner
Em 07/06/2011 11:29, Rodrigo Renji rodrigo.uff.m...@gmail.com escreveu:

 Olá!

 Então acho bem bacana esse também ( e nem é tão complicado de
 demonstrar, eu acho )


 Esse critério pode ser usado para estudar a convergência de [ SOMA de
 1/ k^p  ]  também

 pois  [ SOMA de 2^k / 2^(kp)  ]  =  [ SOMA de 2^(k (1-p))  ]

 se 1 - p 0, isto é 1 p a série converge por série geometrica

 se 1-p  0 , 1  p a série diverge de novo por série geometrica .

 =
 Instruções para entrar na lista, sair da lista e usar a lista em
 http://www.mat.puc-rio.br/~obmlistas/obm-l.html
 =


[obm-l] Re: [obm-l] RE: [obm-l] Re: [obm-l] Re: [obm-l] dúvida sobre séries

2011-06-07 Por tôpico Rodrigo Renji
Olá!

Então acho bem bacana esse também ( e nem é tão complicado de
demonstrar, eu acho )


Esse critério pode ser usado para estudar a convergência de [ SOMA de
1/ k^p  ]  também

pois  [ SOMA de 2^k / 2^(kp)  ]  =  [ SOMA de 2^(k (1-p))  ]

se 1 - p 0, isto é 1 p a série converge por série geometrica

se 1-p  0 , 1  p a série diverge de novo por série geometrica .

=
Instruções para entrar na lista, sair da lista e usar a lista em
http://www.mat.puc-rio.br/~obmlistas/obm-l.html
=


[obm-l] RE: [obm-l] Re:[obm-l] RE: [obm-l] Re:[obm-l] dúvida chara!

2004-05-23 Por tôpico Rogério Moraes de Carvalho
Olá Osvaldo,

Não há a necessidade de formalidades, mesmo porque eu não sou Dr..
Eu gostaria de ressaltar o seguinte comentário que eu coloquei no início dos
meus comentários, caso não tenha ficado claro: A análise que eu apresento a
seguir corresponde a uma crítica de CARÁTER CONSTRUTIVO com relação à
resolução apresentada pelo Osvaldo. O objetivo desta análise não é depreciar
a resolução do Osvaldo, mas sim de mostrar que é necessário sermos rigorosos
nas resoluções de problemas de Matemática para não chegarmos a resultados 
incorretos. Muitas vezes podemos encontrar uma resposta correta para uma
questão resolvendo-a de maneira errada.

É importante ficar claro que mesmo para o conjunto dos números
inteiros a sua solução está incompleta. Observe que você não verificou se os
valores de a e b encontrados produzem valores inteiros de x e y.
Neste caso, você não utilizaria a condição a = b  0 e encontraria todos os
valores inteiros de a e b que reproduzem o produto. Veja o exemplo da
questão modificada para constatar que a sua resolução apresentará resultados
inválidos mesmo no conjunto dos números inteiros.

Atenciosamente,

Rogério Moraes de Carvalho
-Original Message-
From: [EMAIL PROTECTED] [mailto:[EMAIL PROTECTED] On
Behalf Of Osvaldo
Sent: domingo, 23 de maio de 2004 17:54
To: obm-l
Subject: [obm-l] Re:[obm-l] RE: [obm-l] Re:[obm-l] dúvida chara!

Desculpe-me se fui parcial Dr., porém equivoquei-me ao 
ler o enunciado da questão. Eu apenas fiz os calculos 
para os números inteiros e não naturais, ou seja, 
inclui algumas possibilidades a mais. 
Obrigado pela observação!





 Olá colegas da lista,
 
   Apesar da resolução apresentada pelo Osvaldo 
ter seguido um possível
 raciocínio correto para resolver esta questão, a 
análise dele está
 incompleta porque omite alguns passos muito 
importantes, o que pode nos
 levar a encontrar soluções inválidas. Neste problema 
especificamente, a
 resposta encontrada está correta, porém, se 
modificarmos o valor da
 diferença de quadrados de 27 para outro valor, então 
a resolução dele pode
 nos levar a resultados errados.
 
   A análise que eu apresento a seguir 
corresponde a uma crítica de
 caráter construtivo com relação à resolução 
apresentada pelo Osvaldo. O
 objetivo desta análise não é depreciar a resolução 
do Osvaldo, mas sim de
 mostrar que é necessário sermos rigorosos nas 
resoluções de problemas de
 Matemática para não chegarmos a resultados 
incorretos. Muitas vezes podemos
 encontrar uma resposta correta para uma questão 
resolvendo-a de maneira
 errada.
 
   Na resolução apresentada abaixo, considere 
que = significa
 implica e = significa maior ou igual a.
 
 
 QUESTÃO ORIGINAL:
 
 A diferença entre os quadrados de dois números 
naturais é 27. UM dos
 possíveis valores do quadrado da soma desses dois 
números:
 a)529
 b)625
 c)729
 d)841
 
 
 RESOLUÇÃO POSSÍVEL:
 
 Sejam x e y os dois números naturais, então devemos 
ter:
 x^2 - y^2 = 27 = (x + y)(x - y) = 27
 
 Adotando a = x + y e b = x - y, teremos:
 a.b = 27 (i) (Observe que o produto de a e b é 
positivo)
 Resolvendo o sistema de equações nas variáveis x e 
y, podemos encontrar x e
 y em função de a e b:
 a + b = (x + y) + (x - y) = a + b = 2x = x = (a 
+ b)/2 (ii)
 a - b = (x + y) - (x - y) = a - b = 2y = y = (a -
 b)/2 (iii)
 
 Como x e y são naturais, então x = 0 e y = 0. 
Portanto:
 x + y = 0 + 0 = a = 0. De acordo com a igualdade 
(i), a não pode ser 0,
 logo a  0 (iv)
 Como a.b  0 (i) e a  0 (iv), então b  0 (v)
 y = 0 = -y = 0 = y = 0 e 0 = -y = y = -y = 
x + y = x - y =
 a = b (vi)
 Por (v) e (vi), concluímos que: a = b  0 (vii)
 
 Sendo assim, devemos encontrar a e b inteiros tais 
que sejam satisfeitas as
 seguintes condições:
 a.b = 27 (ii)
 a = b  0 (vii)
 x = (a + b)/2 (ii) seja um número natural.
 y = (a - b)/2 (iii) seja um número natural.
 
 Analisando os divisores de 27, podemos concluir que 
existem apenas dois
 pares de valores de a e b que satisfazem as 
condições (ii) e (vii):
 (a = 27 e b = 1) ou (a = 9 e b = 3)
 
 Para a = 27 e b = 1:
 x = (27 + 1)/2 = 14 é um número natural.
 y = (27 - 1)/2 = 13 é um número natural.
 Portanto, x = 14 e y = 13 é uma solução possível.
 
 Para a = 9 e b = 3:
 x = (9 + 3)/2 = 6 é um número natural.
 y = (9 - 3)/2 = 3 é um número natural.
 Portanto, x = 6 e y = 3 é uma solução possível.
 
 Possíveis valores para (x + y)^2:
 (x + y)^2 = (14 + 13)^2 = 27^2 = 729
 (x + y)^2 = (6 + 3)^2 = 9^2 = 81
 
 Resposta: Alternativa c
 
 
 Observação: Pode parecer que os passos apresentados 
para deduzir as
 condições são desnecessários, mas são eles que 
garantem a validade das
 soluções encontradas.
 
 
 EXPLICAÇÃO DO MOTIVO DA RESOLUÇÃO APRESENTADA PELO 
OSVALDO SER INCOMPLETA:
 
 Na resolução são apresentados 4 valores possíveis 
para a e b (a,b):
 {(1,27),(3,9),(9,3),(27,1)}. Porém, (1,27) e (3,9) 
não satisfazem a condição
 (vii): a = b  0. Portanto, somente os pares (9,3) 
e (27,1) correspondem a
 possíveis 

[obm-l] Re: [obm-l] Re: [obm-l] Re: [obm-l] Re: [obm-l] dúvida

2002-10-01 Por tôpico Henrique Branco

Eder, desculpe, acho que minha pergunta que foi mal colocada.
A questão é justamente por que contar da esquerda para a direita?
Já que a 610 na base 2 é 1001100010, por que não contar como se ele tivesse
acertado a primeira, a quarta, quinta...? (apesar de que isso não satisfaria
a questão, pois geraria uma resposta errada)

Grato,
Henrique.

- Original Message -
From: Eder [EMAIL PROTECTED]
To: [EMAIL PROTECTED]
Sent: Tuesday, October 01, 2002 1:54 PM
Subject: [obm-l] Re: [obm-l] Re: [obm-l] Re: [obm-l] dúvida


 0010 0100 0001,mas não aparecem 0110 ou 0010 novamente
 O que isso tem de especial?

 Na hora de somarmos as pontuações,os 1's aparecerão  na ordem das
perguntas
 respondidas corretamente!De fato,basta lembrar como se soma números
 binários,como teremos sempre 1 em cima de zero ou zero em cima de
 zero

 Pois é,basta representar o resultado na base 2 e contar da direita para
 esquerda,essa é a ordem das perguntas.Daí ele ter respondido as perguntas
 2,6,7 e 10.Note que a representação na base 2 não compromete,ou melhor,
ele
 está por trás de tudo!Caso a pergunta 1 tivesse sido respondida
corretamente
 o candidato ganharia 1*2º=1 ponto,conforme expresso no enunciado.Espero
ter
 ajudado.


=
Instruções para entrar na lista, sair da lista e usar a lista em
http://www.mat.puc-rio.br/~nicolau/olimp/obm-l.html
O administrador desta lista é [EMAIL PROTECTED]
=



[obm-l] Re: [obm-l] Re: [obm-l] Re: [obm-l] Re: [obm-l] Re: [obm-l] dúvida

2002-10-01 Por tôpico Douglas Carvalho

Pois vc representa os numeros com as ordens
crescendo da dir para esq.
Por exemplo:

512 (decimal)

é 2 * 10^0 (unidade)
+
   1 * 10^1 (dezena)
+
   5 * 10^2 (centena)

assim como

10100 (binário)
é:

0 * 2^0 +
0 * 2^1 +
1 * 2^2 +
0 * 2^3 +
1 * 2^4 =
 20 em decimal.

Assim, no problema proposto, o 2^0 seria
a primeira questão, 2^1 seria a 2ª e assim
por diante.

Amplexos,

Douglas

-Mensagem Original-
De: Henrique Branco [EMAIL PROTECTED]
Para: [EMAIL PROTECTED]
Enviada em: Terça-feira, 1 de Outubro de 2002 19:35
Assunto: [obm-l] Re: [obm-l] Re: [obm-l] Re: [obm-l] Re: [obm-l] dúvida


 Eder, desculpe, acho que minha pergunta que foi mal colocada.
 A questão é justamente por que contar da esquerda para a direita?
 Já que a 610 na base 2 é 1001100010, por que não contar como se ele
tivesse
 acertado a primeira, a quarta, quinta...? (apesar de que isso não
satisfaria
 a questão, pois geraria uma resposta errada)

 Grato,
 Henrique.

 - Original Message -
 From: Eder [EMAIL PROTECTED]
 To: [EMAIL PROTECTED]
 Sent: Tuesday, October 01, 2002 1:54 PM
 Subject: [obm-l] Re: [obm-l] Re: [obm-l] Re: [obm-l] dúvida


  0010 0100 0001,mas não aparecem 0110 ou 0010 novamente
  O que isso tem de especial?
 
  Na hora de somarmos as pontuações,os 1's aparecerão  na ordem das
 perguntas
  respondidas corretamente!De fato,basta lembrar como se soma números
  binários,como teremos sempre 1 em cima de zero ou zero em cima de
  zero
 
  Pois é,basta representar o resultado na base 2 e contar da direita para
  esquerda,essa é a ordem das perguntas.Daí ele ter respondido as
perguntas
  2,6,7 e 10.Note que a representação na base 2 não compromete,ou melhor,
 ele
  está por trás de tudo!Caso a pergunta 1 tivesse sido respondida
 corretamente
  o candidato ganharia 1*2º=1 ponto,conforme expresso no enunciado.Espero
 ter
  ajudado.


 =
 Instruções para entrar na lista, sair da lista e usar a lista em
 http://www.mat.puc-rio.br/~nicolau/olimp/obm-l.html
 O administrador desta lista é [EMAIL PROTECTED]
 =


=
Instruções para entrar na lista, sair da lista e usar a lista em
http://www.mat.puc-rio.br/~nicolau/olimp/obm-l.html
O administrador desta lista é [EMAIL PROTECTED]
=